Assuming GCH: if $mathrm{cf}(kappa) leq lambda < kappa$, then $kappa^lambda = kappa^+$ (Jech Theorem 5.15)












1












$begingroup$


I am trying to fill in the details for part (ii) of Theorem 5.15 in Jech's Set Theory:




Theorem 5.15 If GCH holds and $kappa$ and $lambda$ are infinite cardinals, then



(i) If $kappa leq lambda$, then $kappa^lambda = lambda^+$.



(ii) If $mathrm{cf}(kappa) leq lambda < kappa$, then $kappa^lambda = kappa^+$.



(iii) If $lambda < mathrm{cf}(kappa)$, then $kappa^lambda = kappa$.




In the proof of (ii) he just states that it follows from these two lemmas:




Lemma 5.7 If $|A| = kappa geq lambda$, then the set $[A]^lambda$ has cardinality $kappa^lambda.$




(Here $[A]^lambda = {X subset A : |X| = lambda}.$)




Lemma 5.8 If $lambda$ is an infinite cardinal and $kappa_i > 0$ for each $i < lambda$, then



$$sum_{i<lambda}{kappa_i} = lambdacdotsup_{i<lambda}{kappa_i}.$$




I am struggling to construct an explicit proof using these results and a few facts about cardinal arithmetic such as absorption for infinite cardinals and cardinalities of power sets.



Since, assuming GCH, $kappa^+ = 2^kappa = |mathcal{P}(A)|$ for some $A$ with cardinality $kappa$, I thought I could come up with some sequence ${mu_i : i < mathrm{cf}(kappa)}$ such that



$$kappa^+ = 2^kappa = |mathcal{P}(A)| = sum_{i < mathrm{cf}(kappa)} |[A]^{mu_i}| = sum_{i < mathrm{cf}(kappa)} kappa^{mu_i} = mathrm{cf}(kappa)cdotsup_{i<mathrm{cf}(kappa)}kappa^{mu_i} = mathrm{cf}(kappa)cdotkappa^lambda = kappa^lambda$$



Is this approach a good idea? How would I go about finding the appropriate $mu_i$? I suppose it will have to make use of the assumption that $mathrm{cf}(kappa) leq lambda < kappa$, but I don't see how.



I'd much appreciate any hints on how to proceed (either with my suggestion or another way).










share|cite|improve this question











$endgroup$

















    1












    $begingroup$


    I am trying to fill in the details for part (ii) of Theorem 5.15 in Jech's Set Theory:




    Theorem 5.15 If GCH holds and $kappa$ and $lambda$ are infinite cardinals, then



    (i) If $kappa leq lambda$, then $kappa^lambda = lambda^+$.



    (ii) If $mathrm{cf}(kappa) leq lambda < kappa$, then $kappa^lambda = kappa^+$.



    (iii) If $lambda < mathrm{cf}(kappa)$, then $kappa^lambda = kappa$.




    In the proof of (ii) he just states that it follows from these two lemmas:




    Lemma 5.7 If $|A| = kappa geq lambda$, then the set $[A]^lambda$ has cardinality $kappa^lambda.$




    (Here $[A]^lambda = {X subset A : |X| = lambda}.$)




    Lemma 5.8 If $lambda$ is an infinite cardinal and $kappa_i > 0$ for each $i < lambda$, then



    $$sum_{i<lambda}{kappa_i} = lambdacdotsup_{i<lambda}{kappa_i}.$$




    I am struggling to construct an explicit proof using these results and a few facts about cardinal arithmetic such as absorption for infinite cardinals and cardinalities of power sets.



    Since, assuming GCH, $kappa^+ = 2^kappa = |mathcal{P}(A)|$ for some $A$ with cardinality $kappa$, I thought I could come up with some sequence ${mu_i : i < mathrm{cf}(kappa)}$ such that



    $$kappa^+ = 2^kappa = |mathcal{P}(A)| = sum_{i < mathrm{cf}(kappa)} |[A]^{mu_i}| = sum_{i < mathrm{cf}(kappa)} kappa^{mu_i} = mathrm{cf}(kappa)cdotsup_{i<mathrm{cf}(kappa)}kappa^{mu_i} = mathrm{cf}(kappa)cdotkappa^lambda = kappa^lambda$$



    Is this approach a good idea? How would I go about finding the appropriate $mu_i$? I suppose it will have to make use of the assumption that $mathrm{cf}(kappa) leq lambda < kappa$, but I don't see how.



    I'd much appreciate any hints on how to proceed (either with my suggestion or another way).










    share|cite|improve this question











    $endgroup$















      1












      1








      1





      $begingroup$


      I am trying to fill in the details for part (ii) of Theorem 5.15 in Jech's Set Theory:




      Theorem 5.15 If GCH holds and $kappa$ and $lambda$ are infinite cardinals, then



      (i) If $kappa leq lambda$, then $kappa^lambda = lambda^+$.



      (ii) If $mathrm{cf}(kappa) leq lambda < kappa$, then $kappa^lambda = kappa^+$.



      (iii) If $lambda < mathrm{cf}(kappa)$, then $kappa^lambda = kappa$.




      In the proof of (ii) he just states that it follows from these two lemmas:




      Lemma 5.7 If $|A| = kappa geq lambda$, then the set $[A]^lambda$ has cardinality $kappa^lambda.$




      (Here $[A]^lambda = {X subset A : |X| = lambda}.$)




      Lemma 5.8 If $lambda$ is an infinite cardinal and $kappa_i > 0$ for each $i < lambda$, then



      $$sum_{i<lambda}{kappa_i} = lambdacdotsup_{i<lambda}{kappa_i}.$$




      I am struggling to construct an explicit proof using these results and a few facts about cardinal arithmetic such as absorption for infinite cardinals and cardinalities of power sets.



      Since, assuming GCH, $kappa^+ = 2^kappa = |mathcal{P}(A)|$ for some $A$ with cardinality $kappa$, I thought I could come up with some sequence ${mu_i : i < mathrm{cf}(kappa)}$ such that



      $$kappa^+ = 2^kappa = |mathcal{P}(A)| = sum_{i < mathrm{cf}(kappa)} |[A]^{mu_i}| = sum_{i < mathrm{cf}(kappa)} kappa^{mu_i} = mathrm{cf}(kappa)cdotsup_{i<mathrm{cf}(kappa)}kappa^{mu_i} = mathrm{cf}(kappa)cdotkappa^lambda = kappa^lambda$$



      Is this approach a good idea? How would I go about finding the appropriate $mu_i$? I suppose it will have to make use of the assumption that $mathrm{cf}(kappa) leq lambda < kappa$, but I don't see how.



      I'd much appreciate any hints on how to proceed (either with my suggestion or another way).










      share|cite|improve this question











      $endgroup$




      I am trying to fill in the details for part (ii) of Theorem 5.15 in Jech's Set Theory:




      Theorem 5.15 If GCH holds and $kappa$ and $lambda$ are infinite cardinals, then



      (i) If $kappa leq lambda$, then $kappa^lambda = lambda^+$.



      (ii) If $mathrm{cf}(kappa) leq lambda < kappa$, then $kappa^lambda = kappa^+$.



      (iii) If $lambda < mathrm{cf}(kappa)$, then $kappa^lambda = kappa$.




      In the proof of (ii) he just states that it follows from these two lemmas:




      Lemma 5.7 If $|A| = kappa geq lambda$, then the set $[A]^lambda$ has cardinality $kappa^lambda.$




      (Here $[A]^lambda = {X subset A : |X| = lambda}.$)




      Lemma 5.8 If $lambda$ is an infinite cardinal and $kappa_i > 0$ for each $i < lambda$, then



      $$sum_{i<lambda}{kappa_i} = lambdacdotsup_{i<lambda}{kappa_i}.$$




      I am struggling to construct an explicit proof using these results and a few facts about cardinal arithmetic such as absorption for infinite cardinals and cardinalities of power sets.



      Since, assuming GCH, $kappa^+ = 2^kappa = |mathcal{P}(A)|$ for some $A$ with cardinality $kappa$, I thought I could come up with some sequence ${mu_i : i < mathrm{cf}(kappa)}$ such that



      $$kappa^+ = 2^kappa = |mathcal{P}(A)| = sum_{i < mathrm{cf}(kappa)} |[A]^{mu_i}| = sum_{i < mathrm{cf}(kappa)} kappa^{mu_i} = mathrm{cf}(kappa)cdotsup_{i<mathrm{cf}(kappa)}kappa^{mu_i} = mathrm{cf}(kappa)cdotkappa^lambda = kappa^lambda$$



      Is this approach a good idea? How would I go about finding the appropriate $mu_i$? I suppose it will have to make use of the assumption that $mathrm{cf}(kappa) leq lambda < kappa$, but I don't see how.



      I'd much appreciate any hints on how to proceed (either with my suggestion or another way).







      set-theory cardinals






      share|cite|improve this question















      share|cite|improve this question













      share|cite|improve this question




      share|cite|improve this question








      edited Dec 4 '18 at 16:39









      Asaf Karagila

      302k32427757




      302k32427757










      asked Dec 4 '18 at 16:35









      ryan221bryan221b

      9510




      9510






















          1 Answer
          1






          active

          oldest

          votes


















          1












          $begingroup$

          You fell into a trap that also caught me when I was just starting to read mathematical books. When you see a reference to (5.7) it does not mean Lemma 5.7, but rather the displayed formula whose tag is 5.7, which in this case is the formula:



          $$kappaleqkappa^lambdaleq 2^kappa.$$ Similarly, (5.8) refers to the inequality $$kappa<kappa^lambdaquadtext{ if }lambdageqoperatorname{cf}kappa.$$



          And indeed these are the inequalities needed to prove (i) and (ii).






          share|cite|improve this answer









          $endgroup$













          • $begingroup$
            Oh! That makes a lot more of sense. Thanks! I think it was more confusing as the line immediately above cites Lemma 5.6, then it cites (5.7) and (5.8).
            $endgroup$
            – ryan221b
            Dec 4 '18 at 16:55











          Your Answer





          StackExchange.ifUsing("editor", function () {
          return StackExchange.using("mathjaxEditing", function () {
          StackExchange.MarkdownEditor.creationCallbacks.add(function (editor, postfix) {
          StackExchange.mathjaxEditing.prepareWmdForMathJax(editor, postfix, [["$", "$"], ["\\(","\\)"]]);
          });
          });
          }, "mathjax-editing");

          StackExchange.ready(function() {
          var channelOptions = {
          tags: "".split(" "),
          id: "69"
          };
          initTagRenderer("".split(" "), "".split(" "), channelOptions);

          StackExchange.using("externalEditor", function() {
          // Have to fire editor after snippets, if snippets enabled
          if (StackExchange.settings.snippets.snippetsEnabled) {
          StackExchange.using("snippets", function() {
          createEditor();
          });
          }
          else {
          createEditor();
          }
          });

          function createEditor() {
          StackExchange.prepareEditor({
          heartbeatType: 'answer',
          autoActivateHeartbeat: false,
          convertImagesToLinks: true,
          noModals: true,
          showLowRepImageUploadWarning: true,
          reputationToPostImages: 10,
          bindNavPrevention: true,
          postfix: "",
          imageUploader: {
          brandingHtml: "Powered by u003ca class="icon-imgur-white" href="https://imgur.com/"u003eu003c/au003e",
          contentPolicyHtml: "User contributions licensed under u003ca href="https://creativecommons.org/licenses/by-sa/3.0/"u003ecc by-sa 3.0 with attribution requiredu003c/au003e u003ca href="https://stackoverflow.com/legal/content-policy"u003e(content policy)u003c/au003e",
          allowUrls: true
          },
          noCode: true, onDemand: true,
          discardSelector: ".discard-answer"
          ,immediatelyShowMarkdownHelp:true
          });


          }
          });














          draft saved

          draft discarded


















          StackExchange.ready(
          function () {
          StackExchange.openid.initPostLogin('.new-post-login', 'https%3a%2f%2fmath.stackexchange.com%2fquestions%2f3025799%2fassuming-gch-if-mathrmcf-kappa-leq-lambda-kappa-then-kappa-lamb%23new-answer', 'question_page');
          }
          );

          Post as a guest















          Required, but never shown

























          1 Answer
          1






          active

          oldest

          votes








          1 Answer
          1






          active

          oldest

          votes









          active

          oldest

          votes






          active

          oldest

          votes









          1












          $begingroup$

          You fell into a trap that also caught me when I was just starting to read mathematical books. When you see a reference to (5.7) it does not mean Lemma 5.7, but rather the displayed formula whose tag is 5.7, which in this case is the formula:



          $$kappaleqkappa^lambdaleq 2^kappa.$$ Similarly, (5.8) refers to the inequality $$kappa<kappa^lambdaquadtext{ if }lambdageqoperatorname{cf}kappa.$$



          And indeed these are the inequalities needed to prove (i) and (ii).






          share|cite|improve this answer









          $endgroup$













          • $begingroup$
            Oh! That makes a lot more of sense. Thanks! I think it was more confusing as the line immediately above cites Lemma 5.6, then it cites (5.7) and (5.8).
            $endgroup$
            – ryan221b
            Dec 4 '18 at 16:55
















          1












          $begingroup$

          You fell into a trap that also caught me when I was just starting to read mathematical books. When you see a reference to (5.7) it does not mean Lemma 5.7, but rather the displayed formula whose tag is 5.7, which in this case is the formula:



          $$kappaleqkappa^lambdaleq 2^kappa.$$ Similarly, (5.8) refers to the inequality $$kappa<kappa^lambdaquadtext{ if }lambdageqoperatorname{cf}kappa.$$



          And indeed these are the inequalities needed to prove (i) and (ii).






          share|cite|improve this answer









          $endgroup$













          • $begingroup$
            Oh! That makes a lot more of sense. Thanks! I think it was more confusing as the line immediately above cites Lemma 5.6, then it cites (5.7) and (5.8).
            $endgroup$
            – ryan221b
            Dec 4 '18 at 16:55














          1












          1








          1





          $begingroup$

          You fell into a trap that also caught me when I was just starting to read mathematical books. When you see a reference to (5.7) it does not mean Lemma 5.7, but rather the displayed formula whose tag is 5.7, which in this case is the formula:



          $$kappaleqkappa^lambdaleq 2^kappa.$$ Similarly, (5.8) refers to the inequality $$kappa<kappa^lambdaquadtext{ if }lambdageqoperatorname{cf}kappa.$$



          And indeed these are the inequalities needed to prove (i) and (ii).






          share|cite|improve this answer









          $endgroup$



          You fell into a trap that also caught me when I was just starting to read mathematical books. When you see a reference to (5.7) it does not mean Lemma 5.7, but rather the displayed formula whose tag is 5.7, which in this case is the formula:



          $$kappaleqkappa^lambdaleq 2^kappa.$$ Similarly, (5.8) refers to the inequality $$kappa<kappa^lambdaquadtext{ if }lambdageqoperatorname{cf}kappa.$$



          And indeed these are the inequalities needed to prove (i) and (ii).







          share|cite|improve this answer












          share|cite|improve this answer



          share|cite|improve this answer










          answered Dec 4 '18 at 16:44









          Asaf KaragilaAsaf Karagila

          302k32427757




          302k32427757












          • $begingroup$
            Oh! That makes a lot more of sense. Thanks! I think it was more confusing as the line immediately above cites Lemma 5.6, then it cites (5.7) and (5.8).
            $endgroup$
            – ryan221b
            Dec 4 '18 at 16:55


















          • $begingroup$
            Oh! That makes a lot more of sense. Thanks! I think it was more confusing as the line immediately above cites Lemma 5.6, then it cites (5.7) and (5.8).
            $endgroup$
            – ryan221b
            Dec 4 '18 at 16:55
















          $begingroup$
          Oh! That makes a lot more of sense. Thanks! I think it was more confusing as the line immediately above cites Lemma 5.6, then it cites (5.7) and (5.8).
          $endgroup$
          – ryan221b
          Dec 4 '18 at 16:55




          $begingroup$
          Oh! That makes a lot more of sense. Thanks! I think it was more confusing as the line immediately above cites Lemma 5.6, then it cites (5.7) and (5.8).
          $endgroup$
          – ryan221b
          Dec 4 '18 at 16:55


















          draft saved

          draft discarded




















































          Thanks for contributing an answer to Mathematics Stack Exchange!


          • Please be sure to answer the question. Provide details and share your research!

          But avoid



          • Asking for help, clarification, or responding to other answers.

          • Making statements based on opinion; back them up with references or personal experience.


          Use MathJax to format equations. MathJax reference.


          To learn more, see our tips on writing great answers.




          draft saved


          draft discarded














          StackExchange.ready(
          function () {
          StackExchange.openid.initPostLogin('.new-post-login', 'https%3a%2f%2fmath.stackexchange.com%2fquestions%2f3025799%2fassuming-gch-if-mathrmcf-kappa-leq-lambda-kappa-then-kappa-lamb%23new-answer', 'question_page');
          }
          );

          Post as a guest















          Required, but never shown





















































          Required, but never shown














          Required, but never shown












          Required, but never shown







          Required, but never shown

































          Required, but never shown














          Required, but never shown












          Required, but never shown







          Required, but never shown







          Popular posts from this blog

          To store a contact into the json file from server.js file using a class in NodeJS

          Redirect URL with Chrome Remote Debugging Android Devices

          Dieringhausen